Rigorous Multivariable Limit Definition Problem

Click For Summary
The discussion revolves around proving the rigorous definition of a limit for the function f(x,y) as (x,y) approaches (1,1). The user initially struggles with the denominator and expresses uncertainty about working with limits not centered at (0,0). A typo in the function is identified, clarifying that (x/1)^2 should be (x-1)^2. Participants suggest shifting the coordinates to simplify the problem and emphasize the need to derive δ from ε in the limit definition. Overall, the conversation focuses on strategies for applying the ε-δ definition to this specific limit problem.
dumbdumNotSmart
Messages
41
Reaction score
3

Homework Statement


Hey I'm trying to prove the rigorous definition of limit for the following function:
Lim (x,y) approaches (1,1) of f(x,y)=(y*(x-1)^(4/3))/((x/1)^2+abs(x)*y^2)

Homework Equations


abs(x^2)<abs(x^2 +y^2)

The Attempt at a Solution


I know the rigorous definition of limit. I tried to constraint the denominator by eliminating one of the terms since both are greater than zero, however, I was left with what seemed like terms that could not be constrained. I never really done a rigorous definition of limit that's not centered on (0,0). I would appreciate some help.
 
Physics news on Phys.org
If you feel more comfortable with limits centred at ##(0,0)##, you can deform the problem (in order to get an idea for ##\delta##), by doing a shift ##(x,y)\rightarrow (x-1,y-1)##. I'm assuming proofs are required to be directly from the ##\epsilon##-##\delta## definition?
 
dumbdumNotSmart said:

Homework Statement


Hey I'm trying to prove the rigorous definition of limit for the following function:
Lim (x,y) approaches (1,1) of f(x,y)=(y*(x-1)^(4/3))/((x/1)^2+abs(x)*y^2)
Typo above? Should the (x/1)^2 be (x - 1)^2?
dumbdumNotSmart said:

Homework Equations


abs(x^2)<abs(x^2 +y^2)

The Attempt at a Solution


I know the rigorous definition of limit. I tried to constraint the denominator by eliminating one of the terms since both are greater than zero, however, I was left with what seemed like terms that could not be constrained. I never really done a rigorous definition of limit that's not centered on (0,0). I would appreciate some help.
 
Mark44 said:
Typo above? Should the (x/1)^2 be (x - 1)^2?
Yes, that is in fact a typo.
 
Lucas SV said:
If you feel more comfortable with limits centred at ##(0,0)##, you can deform the problem (in order to get an idea for ##\delta##), by doing a shift ##(x,y)\rightarrow (x-1,y-1)##. I'm assuming proofs are required to be directly from the ##\epsilon##-##\delta## definition?
That's right, however I may constraint the whole thing backwards, that is, work from ε and thereafter find δ in the form of ||(x,y)-(v,w)|| inside the ε I am constraining.(which in the definition of the limit is |f(x,y)-L|<ε L being the Limit I am trying to prove)... I'm not sure how much sense that made, let me know if you need any more clarification. The course is not in english, as you might have noticed.
 
Question: A clock's minute hand has length 4 and its hour hand has length 3. What is the distance between the tips at the moment when it is increasing most rapidly?(Putnam Exam Question) Answer: Making assumption that both the hands moves at constant angular velocities, the answer is ## \sqrt{7} .## But don't you think this assumption is somewhat doubtful and wrong?

Similar threads

  • · Replies 1 ·
Replies
1
Views
2K
  • · Replies 6 ·
Replies
6
Views
2K
  • · Replies 5 ·
Replies
5
Views
1K
  • · Replies 4 ·
Replies
4
Views
2K
  • · Replies 12 ·
Replies
12
Views
2K
  • · Replies 6 ·
Replies
6
Views
2K
  • · Replies 6 ·
Replies
6
Views
2K
  • · Replies 7 ·
Replies
7
Views
2K
  • · Replies 8 ·
Replies
8
Views
2K
Replies
9
Views
2K